Luận văn Đa thức trong các bài toán thi học sinh giỏi

Danh sách kí hiệu 3

Mở đầu 4

Chương 1. Đa thức một biến 7

1.1 Định nghĩa và các tính chất . . . . . . . . . . . . . . . . . . . . . 7

1.1.1 Định nghĩa . . . . . . . . . . . . . . . . . . . . . . . . . 7

1.1.2 Các phép tính trên đa thức . . . . . . . . . . . . . . . . . 8

1.1.3 Các tính chất cơ bản . . . . . . . . . . . . . . . . . . . . 9

1.2 Phép chia đa thức. Ước chung lớn nhất và nhỏ nhất . . . . . . . . 11

1.2.1 Phép chia đa thức . . . . . . . . . . . . . . . . . . . . . . 11

1.2.2 Thuật toán Euclide . . . . . . . . . . . . . . . . . . . . . 11

1.3 Nghiệm của đa thức. Phương trình bậc cao . . . . . . . . . . . . . 16

1.3.1 Nghiệm của đa thức . . . . . . . . . . . . . . . . . . . . 16

1.3.2 Phương trình bậc cao . . . . . . . . . . . . . . . . . . . . 22

1.4 Đạo hàm của đa thức. Định lý Taylor . . . . . . . . . . . . . . . . 32

Chương 2. Đa thức bất khả quy 36

2.1 Đa thức bất khả quy . . . . . . . . . . . . . . . . . . . . . . . . . 36

2.1.1 Đa thức với hệ số thực và phức . . . . . . . . . . . . . . . 37

2.1.2 Đa thức bất khả quy của vành Q[x] . . . . . . . . . . . . . 40

pdf66 trang | Chia sẻ: honganh20 | Ngày: 28/02/2022 | Lượt xem: 376 | Lượt tải: 1download
Bạn đang xem trước 20 trang tài liệu Luận văn Đa thức trong các bài toán thi học sinh giỏi, để xem tài liệu hoàn chỉnh bạn click vào nút DOWNLOAD ở trên
ới i= 1,2,3, . . . Chứng minh rằng Pn(x) = x có 2n nghiệm thực phân biệt. Lời giải. Ta thu hẹp việc xét nghiệm của phương trình trên đoạn −2≤ x≤ 2. Đặt x= 2cos(t). Khi đó, bằng quy nạp ta chứng minh được Pn(x) = 2cos(2nt). Thêm nữa, phương trình Pn(x) = x trở thành cos(2nt) = cos(t). Từ đó ta được 2n nghiệm t = 2kpi 2n−1 , t = 2kpi 2n+1 , k = 1,2, . . . ,n Vậy phương trình Pn(x) = x có 2n nghiệm thực phân biệt. Bài toán 1.3.5. Cho đa thức f (x) = a0+a1x+ . . .+anxn có n nghiệm thực. Chứng minh với mọi p> n−1 thì đa thức g(x) = a0+a1.p.x+a2.p(p−1)x2+ . . .+anp(p−1) . . .(p−n+1)xn. cũng có n nghiệm thực. Lời giải. Để giải bài toán ta xét hai trường hợp Trường hợp 1. Đa thức f (x) không nhận x= 0 làm nghiệm. Ta chứng minh bằng quy nạp. Với n= 1 bài toán hiển nhiên đúng . 20 Giả sử đúng với n= k, ta chứng minh đúng với n= k+1, tức là nếu đa thức f (x) = a0+a1x+ . . .+ak+1xk+1 có k+1 nghiệm thực khác 0 thì đa thức g(x) = a0+ pa1x+ . . .+ p(p−1) . . .(p− k)ak+1xk+1 cũng có k+1 nghiệm thực khác 0 với mọi p> k. Gọi c là một nghiệm của f (x) thì f (x) = (x− c)q(x), (1.8) với q(x) là một đa thức nào đó là đa thức bậc k của x, q(x) = b0+b1x+ . . .+bkxk. (1.9) Thay (1.9) vào (1.8) rồi đồng nhất hệ số ta được a0 = cb0, a1 = cb1+b0, . . . ak = cbk+bk−1, ak+1 = bk Do đó g(x) = a0+ pa1x+ . . .+ p(p−1) . . .(p− k)ak+1xk+1 = cb0+ p(cb1+b0)x+ . . .+ p(p−1)(p− k)bkxk+1 = cQ(x)+ pxQ(x)− x2Q(x) trong đó Q(x) = b0+b1px+ . . .+ p(p−1) . . .(p− k+1)bkxk. Do f (x) có k+1 nghiệm thực khác 0 nên q(x) có k nghiệm thực khác 0. Mặt khác p> k nên p> k−1. Cho nên theo giả thiết quy nạp ta có đa thức Q(x) có k nghiệm thực. Do đó g(x) có k+1 nghiệm thực. Vậy theo nguyên lý quy nạp, bài toán đúng. Trường hợp 2. Đa thức f (x) nhận x= 0 làm nghiệm. 21 Giả sử x= 0 là nghiệm bội k của f (x), với k ∈ Z+, k ≤ n. Khi đó ta có f (x) = akxk+ . . .+anxn = (anxn−k+ . . .+ak)xk và g(x) = p(p−1) . . .(p− k+1)ak.xk+ . . .+ p(p−1) . . .(p−n+1)an.xn = p(p−1) . . .(p− k+1)xk[ak+ . . .+(p− k) . . .(p−n+1)anxn−k]. Vì f (x) có n nghiệm thực nên H(x) = ak+ . . .+ anxn−k có (n− k) nghiệm thực khác 0. Do đó áp dụng kết quả của Trường hợp 1 cho H(x) và p′ = p− k > n− k−1 (do p> n−1), ta được đa thức R(x) = ak+ . . .+(p− k) . . .(p−n+1)anxn−k có n− k nghiệm thực. Vậy g(x) có n nghiệm thực. Bài toán 1.3.6 (Trung Quốc 1996). Cho đa thức p(x) bậc 5 có 5 nghiệm thực phân biệt. Tìm số bé nhất của các hệ số khác 0. Lời giải. Xét p(x) = ax5+bx4+ cx3+dx+ e, a 6= 0. Nếu có 4 hệ số bằng 0 thì b= c= d = e= 0 nên p(x) = ax5 có nghiệm bội (loại) tức là p(x) không thể có một hệ số khác 0. Do đó p(x) có ít nhất hai hệ số khác 0. Xét p(x) = ax5+bxn, n≥ 2 thì p(x) có nghiệm bội. Ta tiếp tục loại trường hợp này. Xét p(x) = ax5+dx= ax ( ax4+ ad ) có tối đa ba nghiệm. Ta cũng loại. Xét p(x) = ax5+ e có một nghiệm. Ta cũng loại. Do đó p(x) có ít nhất ba hệ số khác 0. Chọn p(x) = x5−5x3+4x= x(x2−4) thì p(x) có đúng 5 nghiệm phân biệt và đúng ba hệ số khác 0. Vậy số bé nhất của hệ số khác 0 là 3. 22 1.3.2 Phương trình bậc cao Lý thuyết giải phương trình bậc 3 tổng quát Xét phương trình đa thức bậc ba ax3+bx2+ cx+d = 0, a 6= 0. (1.10) Ngoài việc tách nhóm số hạng hoặc tìm một nghiệm rồi phân tích nhân tử, dùng hằng đẳng thức, ta có cách giải tổng quát như sau: Trước hết, chia 2 vế cho a 6= 0 đưa về phương trình: x3+Bx2+Cx+D = 0. Tiếp theo đặt x= y− B3 đưa tiếp về phương trình: y3− py= q, trong đó p= b2 3 −C, q=−2B 3 27 + BC 3 −D. Có hai hướng để giải phương trình y3− py= q. (1.11) Hướng thứ nhất. Đặt y= u+ v và chọn u,v= p3 thì từ y 3 = u3+ v3+3uv(u+ v) ta có u3+ v3 = q và u3.v3 = P 3 27 . Vậy u 3,v3 là nghiệm phương trình Z3−qZ+ p 3 27 = 0. Nếu ∆< 0 sau này ta dùng số phức để giải quyết. Hướng thứ hai. Nếu p= 0 thì (1.11) tương đương với y3 = q, tức là y= √ q. Nếu p> 0. Đặt y= 2 √ p 3 t thì (1.11) tương đương với 4t3−3t = m (1.12) trong đó m= 3 √ 3q 2p √ p . Xét |m| ≤ 1, đặt m= cosα thì (1.12) có ba nghiệm t1 = cos α 3 , t2 = cos α+2pi 3 , t3 = cos α−2pi 3 . 23 Xét |m| > 1, đặt m = 12 ( d3+ 1d3 ) suy ra d3 = m±√m2−1. Phương trình (1.12) có một nghiệm t = 1 2 ( d+ 1 d ) = 1 2 ( 3 √ m+ √ m2−1+ 3 √ m− √ m2−1 ) Nếu p< 0. Đặt y= 2. √ − p3 t thì (1.11) tương đương với 4t3+3t = m. (1.13) Ta đặt tiếp m= 1 2 ( k3− 1 k3 ) suy ra k3 = m± √ m2+1. Phương trình (1.13) có một nghiệm t = 1 2 ( k− 1 k ) = 1 2 ( 3 √ m+ √ m2+1+ 3 √ m− √ m2+1 ) . Ta gọi các phương trình bậc ba 4x3+3x+−m= 0, 4x3−3x−m= 0 là các dạng phương trình bậc 3 chuẩn tắc. Ý nghĩa cơ bản là mọi phương trình bậc 3 đều đưa về được dạng chuẩn tắc đó. Chú ý thêm khi |m| ≥ 1, 4x3+3x−m= (x−α)(4x2+4αx+4α2+3). Với α = 1 2 ( 3 √ m+ √ m2+1+ 3 √ m− √ m2+1 ) có ∆′ =−12(α2+1)< 0 và 4x3−3x−m= (x−β )(4x2+4βx+4β 2+3). Với β = 1 2 ( 3 √ m+ √ m2−1+ 3 √ m− √ m2−1 ) có ∆′ =−12(1−β 2)< 0. 24 Các phương trình bậc 4 đặc biệt 1. Phương trình có dạng ax4+bx2+ c= 0, a 6= 0. (1.14) Đặt t = x2, t ≤ 0 thì đưa về phương trình bậc hai at2+bt+ c= 0 2. Phương trình có dạng (x+a)4+(x+b)4 = c. (1.15) Đặt t = x+ a+b2 thì đưa về phương trình trùng phương At 4+Bt2+C = 0 và giải như trên. 3. Phương trình có dạng (ax2+bx+ c)(ax2+bx+d) = m. (1.16) Đặt t = ax2+bx thì đưa về phương trình bậc hai (t+ c)(t+d) = m. 4. Phương trình có dạng (x+a)(x+b)(x+ c)(x+d) = m. (1.17) Nếu có a+ d = b+ c thì ghép cặp (x+ a)(x+ d) và (x+ b)(x+ c) rồi đặt t = x2+(a+d)x= x2+(b+ c)x để đưa về dạng trên. 5. Phương trình có dạng ax4+bx3+ cx2+dx+ e= 0. (1.18) Nếu ad2= eb2 6= 0 thì chia hai vế cho x2 6= 0 rồi đặt t = x+ eax (Phương trình quy hồi mở rộng bậc bốn). 25 Phương trình quy hồi (đối xứng hệ số) Xét phương trình a0xn+a1xn−1+a2xn−2+ · · ·+an−2x2+an−1x+an = 0 (1.19) trong đó a0 = an; a1 = an−1; a2 = an−2; . . . Xét n chẵn, n = 2m. Chia hai vế cho xm 6= 0.. Đặt t = x+ 1x , |t| ≥ 2 đưa về phương trình bậc m= n2 . Xét n lẻ, n= 2m+1. Phương trình có nghiệm x=−1 nên phân tích ra thừa số (x+1) và thừa số bậc 2m lại là phương trình quy hồi bậc chẵn như trên. Đôi khi ta mở rộng dạng quy hồi (quy hồi kèm tỉ lệ) với cách đặt t = x− 1 x , t = x+ a x Phương trình bậc cao Xét phương trình a0xn+a1xn−1+ · · ·+an−1x+an = 0, a0 6= 0. (1.20) Nguyên tắc chung là biến đổi về dạng tích, đặt ẩn phụ để đưa về phương trình bậc thấp hơn. Đặc biệt • Nếu tổng các hệ số bằng 0 thì có nghiệm x= 1. • Nếu tổng đan dấu các hệ số bằng 0 thì có nghiệm x=−1 . • Nhắc lại, nghiệm hữu tỉ nếu có thì có dạng x = pq với p | an và q | a0. Thế trực tiếp hoặc dúng sơ đồ Hoocne để thử nghiệm. Đôi khi phương trình bậc cao đối với biến x mà lại bậc thấp đối với tham số thì ta chuyển về phương trình theo ẩn là tham số đó. 26 Bài toán 1.3.7. Giải các phương trình sau (a) 4x3−10x2+6x−1= 0. (b) 8x3−36x+27= 0. Lời giải. (a) Ta có 4x3−10x2+6x−1= 0 ⇔ 4x3−2x2−8x2+4x+2x−1= 0 ⇔ 2x2(2x−1)−4x(2x−1)+(2x−1) = 0 ⇔ (2x−1)(2x2−4x+1) = 0 ⇔ 2x−1= 0 hoặc 2x2−4x+1= 0. Vậy nghiệm của phương trình là x= 1 2 , x= 2±√2 2 . Bài toán 1.3.8. Tìm quan hệ giữa p và q để phương trình x3+ px+ q = 0 có thể viết dưới dạng x4 = (x2−ax+b)2. Áp dụng kết quả đó để giải phương trình: x3−18x+27= 0. Lời giải. Ta có x4− (x2−ax+b)2 =−m(x3+ px+q). Suy ra a2+2b= 0, 2ab= pm, 2a= m,b2 = qm. Từ đó b= p, kéo theo p2 = mq. Vậy m= p 2 q . Mặt khác a2 =−2b⇒ a= m2 ⇒ m 2 4 =−2b=−2p⇒ m2 =−8p. Từ đó p 4 q2 = −8⇒ p4+8pq2 = 0. Vậy quan hệ giữa p và q: p3+8q2 = 0. Ta có x3−18x+27= 0 ⇔ x4 = (x2+6x−18)2 27 ⇔ (6x−18)(2x2+6x−18) = 0 ⇔ x= 3 hoặc x= −3± √ 35 2 . Bài toán 1.3.9. Giải và biện luận phương trình x3−3x2+3(a+1)x− (a+1)2 = 0. (1.21) Lời giải. Ta có x3−3x2+3(a+1)x− (a+1)2 = 0 tương đương với −x3 =−3x2+3(a+1)x− (a+1)2. Nếu a 6=−1 thì nhân hai vế với (a+1), ta được −x3(a+1) =−3x2(a+1)+3(a+1)2x− (a+1)3 Cộng hai vế với x3 ta được −ax3 = (x−a−1)3. Từ đó ta được x−a−1=− 3√ax, suy ra x= a+1 3 √ a+1 = 3√ a2− 3√a. Nếu a=−1 thì dễ thấy phương trình có hai nghiệm x1 = 0 và x2 = 3. Bài toán 1.3.10. Giải các phương trình sau (a) x4−2x3−6x2+16x−8= 0; (b) x4+ x2+4x−3= 0; Lời giải. (a) Ta có x4−2x3−6x2+16x−8= 0 ⇔ (x−2)(x3−6x+4) = 0 ⇔ (x−2)(x−2)(x2+2x−2) = 0 ⇔ (x−2)2(x2+2x−2) = 0. Vậy nghiệm của phương trình là x= 2, x=−1±√3. 28 (b) Ta có x4+ x2+4x−3= 0 ⇔ x4 =−x2−4x+3 ⇔ (x+1)2 = (x−2)2 ⇔ (x2+1)2− (x−2)2 = 0 ⇔ (x2− x+3)(x2+ x−1) = 0 Vậy nghiệm của phương trình là x= −1± √ 5 2 . Bài toán 1.3.11 (IMO 1973). Giả sử phương trình x4+ax3+bx2+ax+1= 0 có nghiệm. Tìm giá trị bé nhất của a2+b2. Lời giải. Gọi x0 là nghiệm của phương trình đã cho, tức là x40+ax 3 0+bx 2 0+ax0+1= 0 Từ đây ta có kết luận x0 6= 0. Chia hai vế cho x20, ta có x20+ax0+b+ a x0 + 1 x20 = 0. Phương trình này tương đương với( x20+ 1 x20 ) +a ( x0+ 1 x0 ) +b= 0. Đặt y= x0+ 1x0 với điều kiện |y|= |x0|+ | 1 x0 | ≥ 2. Ta có (y2−2)+ay+b= 0 Suy ra |2− y2|= |ay+b| ≤ √ a2+b2 √ y2+1⇒ a2+b2 ≥ (2− y 2)2 1+ y2 Đặt t = y2, t ≥ 4. Ta chứng minh (2− t)2 1+ t ≥ 4 5 (1.22) Thật vậy, ta có (1.22) tương đương với 5(2− t)2 ≥ 4(1+ t), tức là 5t2−24t+16≥ 0. Nhưng điều này đúng vì t ≥ 4. Như vậy giá trị bé nhất của a2+b2 là 4/5. 29 Bài toán 1.3.12 (Việt Nam 2002). Giải phương trình√ 4−3√10−3x= x−2 (1.23) Lời giải. Ta có biến đổi tương đương như sau √ 4−3√10−3x= x−2 ⇔ x≥ 24−3√10−3x= (x−2)2 ⇔ x≥ 23√10−3x= 4x− x2 ⇔ x≥ 2, 4x− x 2 ≥ 0 9(10−3x) = (4x− x2)2 ⇔ x≥ 2, 0≤ x≤ 4x4−8x3+16x2+27x−90= 0. ⇔ 2≤ x≤ 4x4−8x3+16x2+27x−90= 0. Bây giờ ta sẽ giải phương trình x4−8x3+16x2+27x−90= 0. (1.24) Bằng cách thử trực tiếp ta thấy x= 3 là một nghiệm nên phương trình (1.24) được viết lại thành (x−3)(x+2)(x2−7x+15) = 0. Vậy phương trình có nghiệm duy nhất x= 3. Bài toán 1.3.13 (Việt Nam 1991). Giải phương trình x3−3x2−8x+40= 8 4√4x+4. (1.25) 30 Lời giải. Từ phương trình x3−3x2−8x+40= 8 4√4x+4 ta có điều kiện x≥−1. Áp dụng bất đẳng thức AM-GM ta có 8 4 √ 4x+4= 4 4 √ 4.4.4.(x+1)≤ x+13. Do đó x3−3x2−8x+40≤ x+13 ⇔ x3−3x2−9x+27≤ 0 ⇔ (x+3)(x2−6x+9)≤ 0 ⇔ (x+3)(x−3)2 ≤ 0. Vì x≥−1 nên (x−3)2 ≥ 0, suy ra x= 3. Thử lại ta thấy đúng. Vậy phương trình có nghiệm duy nhất x= 3. Bài toán 1.3.14. Chứng minh rằng (a) x= 3 √ a+ a+13 √ 8a−1 3 + 3 √ a− a+13 √ 8a−1 3 với a≥ 18 là số tự nhiên (b) 3 √ 2+ 3 √ 4 là số vô tỉ. Lời giải. (a) Áp dụng hằng đẳng thức (u+ v)3 = u3+ v3+3uv(u+ v) ta có x3 = 2a+(1−2a)x ⇔ x3+(2a−1)x−2a= 0 ⇔ (x−1)(x2+ x+2a) = 0. Xét đa thức bậc hai x2+ x+2a có ∆= 1−8a≥ 0 Khi a= 18 , ta có x= 3 √ 1 8+ 3 √ 1 8 = 1 Khi a > 18 , ta có 1− 8a âm nên đa thức x2+ x+ 2a có nghiệm thực duy nhất x= 1. Vậy với a≥ 18 thì x= 3 √ a+ a+1 3 √ 8a−1 3 + 3 √ a− a+1 3 √ 8a−1 3 31 là một số tự nhiên. (b) Đặt x= 3 √ 2+ 3 √ 4. Ta có x3 = 2+4+3 3 √ 8( 3 √ 2+ 3 √ 4) Tức là x3−6x−6. Giả sử x ∈Q. Do hệ số bậc cao nhất của đa thức là 1 nên x là số nguyên. Ta có 2< 3 √ 2+ 3 √ 4< 4 nên x= 3. Do đó x3−6x−6= 3, vô lý. Vậy x là số vô tỷ. Bài toán 1.3.15 (Việt Nam 1984). Tìm đa thức theo x có bậc bé nhất với hệ số nguyên, biết một nghiệm là √ 2+ 3 √ 3. Lời giải. Đặt a= √ 2+ 3 √ 3 ta có a2 = 2+2 √ 2. 3 √ 3+ 3 √ 9 và a3 = 2 √ 2+6 3 √ 3+3 √ 2. 3 √ 9+3 (1.26) Ta rút ra 3√3= a− √ 2, 3√9= a2−2−2 √ 2(a− √ 2) = a2+2−2 √ 2a. Thay vào (1.26) ta có a3 = 2 √ 2+6(a− √ 2)+3 √ 2(a2+2−2 √ 2a)+3, a3+6a−3= √ 2(3a2+2). Bình phương hai vế ta thấy a là nghiệm của đa thức x6−6x4−6x3+12x2−36x+1 Bằng phép đồng nhất hệ số ta chứng minh đa thức trên không phân tích được thành hai đa thức bậc thấp hơn có hệ nguyên nên đa thức trên chính là đa thức có bậc bé nhất thỏa đề bài. 32 1.4 Đạo hàm của đa thức. Định lý Taylor Đạo hàm của một hàm số là một khải niệm thuộc nhánh Giải tích toán học. Trong mục này chúng tôi sẽ trình bày ngắn gọn về đạo hàm của hàm số đa thức. Định nghĩa 1.4.1. Cho f (x) ∈ R[x] và deg f (x) = n, anxn+an−1xn−1+ . . .+a1x+a0 với an 6= 0 • Đa thức nanxn−1+(n−1)an−1xn−2+ . . .+2a2x+a1 được gọi là đạo hàm cấp một của đa thức f (x) (hoặc ngắn gọn là đạo hàm của đa thức f (x)), và được ký hiệu là f ′(x). Bằng quy nạp, ta định nghĩa đạo hàm cấp k của đa thức f (x) là đạo hàm của đạo hàm cấp k− 1. Ta ký hiệu đạo hàm cấp 2, 3 và cấp k tổng quát lần lượt là f ′′(x), f ′′′(x), và f (k)(x). • Với mọi x0 ∈ R, biểu thức sau đây được gọi là khai triển Taylor của đa thức thực f (x) f (x)= f (x0)+ f ′(x0) 1! (x−x0)+. . .+ f (k)(x0) k! (x− x0)k+. . .+ f (n)(x0) n! (x− x0)n. Từ định nghĩa này ta có nếu deg f = n thì deg f ′ = n−1, deg f ′′ = n−2, . . . , deg f (k) = n− k với 1≤ k ≤ n, deg f (x) = 0. Bây giờ ta xét một số bài toán về đạo hàm hàm và khai triển Taylor. Chúng tôi tham khảo [6] trong trình bày. Bài toán 1.4.2 (Ba Lan 1979). Cho đa thức P(x) có bậc n> 1 và có n nghiệm thực x1,x2,x3, . . . ,xn phân biệt. Chứng minh rằng 1 P′(x1) + 1 P′(x2) + . . .+ 1 P′(xn) = 0. 33 Lời giải. Đặt P(x) = a(x− x1)(x− x2) . . .(x− xn),a 6= 0 ta suy ra P′(x) = P1(x)+P2(x)+ . . .+Pn(x) với Pi(x) = n ∏ j=1, j 6=i (x− x j). Ta thấy Pi(x j) = 0 với mọi j 6= i kéo theo P′(x j) = Pj(x j) 6= 0 với mọi j = 1,2, . . . ,n Xét đa thức F(x) = n ∑ i=1 Pi(x) P′(xi) −1 có bậc không vượt quá n−1. Với i= 1,2, . . . ,n ta có F(xi) = Pi(xi)P′(xi)−1= 0 suy ra F(x) có n nghiệm phân biệt. Vậy F(x) = 0. Mặt khác hệ số của F(x) đối với xn−1 bằng 0 nên a P′(x1) + a P′(x2) + . . .+ a P′(xn) = 0. Như vậy 1 P′(x1) + 1 P′(x2) + . . .+ 1 P′(xn) = 0. Bài toán 1.4.3 (CHDCLB Đức 1974). (a) Chứng minh rằng không tồn tại đa thức P(x) để với mọi x ∈R ta có các bất đẳng thức P′(x)> P′′(x), (1.27) P(x)> P′′(x). (1.28) (b) Khẳng định trên còn đúng không nếu thay đổi bất đẳng thức (1.27) bằng bất đẳng thức P(x)> P′(x). (1.29) 34 Lời giải. (a) Nếu P(x) là hằng số thì P′(x) = P′′(x) = 0, và bất đẳng thức (1.27) không thỏa mãn. Giả sử degP(x) = n≥ 1 khi đó nếu n lẻ thì deg(P(x)−P′′(x)) = n là số lẻ, từ đó P(x)−P′′(x)≤ 0 với ít nhất một điểm x ∈ R. Như vậy đối với đa thức P(x) không thỏa mãn hoặc bất đẳng thức (1.28) hoặc bất đẳng thức (1.27). Vậy (a) được chứng minh xong. (b) Chọn P(x) = x2+3. Khi đó với x ∈ R ta có P(x)−P′(x)≡ x2−2x+3> 0 và P(x)−P′′(x)≡ x2+1> 0 nghĩa là khẳng định trên không còn đúng nữa. Bài toán 1.4.4 (Việt Nam 1986). Cho f ∈ R[x] có deg f = n và f (k) = 2k, với k = 0,1,2, . . . ,n. Tính f (n+1). Lời giải. Xét đa thức g(x) = 1+ x 1! + x(x−1) 2! + . . .+ x(x−1)(x−2) . . .(x−n+1) n! . Khi đó degg= n và g(k) = n ∑ i=0 Cik = 2 k = f (k) với n+1 giá trị nên f ≡ g. Do đó f (n+1) = g(n+1) = n ∑ i=0 Cin+1 = 2 n+1−1. Bài toán 1.4.5 (Singapore 1978). Cho đa thức P(x) bậc n và hai số a < b thỏa mãn P(a)< 0,−P′(a)≤ 0, P(a)≤ 0, . . . ,(−1)nP(n)(a)≤ 0, P(b)> 0, P′(b)≥ 0, P(b)≥ 0, . . . ,P(n)(b)≥ 0. Chứng minh các nghiệm thực của P(x) thuộc (a,b). 35 Lời giải. Khai triển Taylor ta có P(x) = P(b)+ P′(b) 1! (x−b)+ P(b) 2! (x−b)2+ . . .+ P (n)(b) n! (x−b)n. Nếu x≥ b thì P(x)> 0 suy ra P(x) không có nghiệm x≥ b. Tương tự P(x) = P(a)+ P′(a) 1! (x−a)+ P(a) 2! (x−a)2+ . . .+ P (n)(a) n! (x−a)n = P(a)+ −P′(a) 1! (a− x)+ P(a) 2! (a− x)2+ . . .+ (−1) n(a) n! (a− x)n Nếu x< a thì P(x)< 0 suy ra P(x) không có nghiệm x≤ a Vậy các nghiệm phải thuộc (a,b). Bài toán được chứng minh xong. Ta gọi ước lượng về nghiệm ở trên là ước lượng Newton. 36 Chương 2 Đa thức bất khả quy 2.1 Đa thức bất khả quy Định nghĩa 2.1.1. Cho đa thức f ∈ Z[x]. Ta gọi đa thức f là bất khả quy trên Z[x] nếu f không phân tích được thành tích hai đa thức thuộc Z[x] với bậc lớn hơn hay bằng 1. Trường hợp f là một đa thức hữu tỷ, ta định nghĩa hoàn toàn tương tự. Mối quan hệ về tính bất khả quy trên Z[x] và Q[x] được phát biểu như sau: Định lí 2.1.2. Nếu đa thức f ∈ Z[x] bất khả quy trên Z[x] thì f cũng bất khả quy trên Q[x]. Để chứng minh định lý này, ta cần bổ đề sau đây. Trước hết, ta gọi đa thức f ∈ Z[x] là nguyên bản nếu các hệ số nguyên tố cùng nhau. Bổ đề 2.1.3 (Bổ đề Gauss). Tích của hai đa thức nguyên bản là một đa thức nguyên bản. Chứng minh. Cho hai đa thức nguyên bản f (x) = a0xn+a1xn−1+ . . .+an, g(x) = b0xm+b1xm−1+ . . .+bm. 37 Khi đó f (x)g(x) = c0xn+m+ c1xn+m−1+ . . .+ cn+m. Giả sử đa thức tích f (x)g(x) không nguyên bản. Khi đó tồn tại một số nguyên tố p là ước chung của các hệ số c0, . . . ,cn+m Vì f nguyên bản nên gọi ai là số đầu tiên mà ai ... p. Vì g nguyên bản nên gọi b j là số đầu tiên mà b j ... p. Bằng cách xét hệ số theo lũy thừa xi+ j ta có hệ số tương ứng không chia hết cho p. Điều này vô lý. Vậy f (x)g(x) là đa thức nguyên bản. Chứng minh Định lý 2.1.2. Cho f ∈ Z[x] là một đa thức bất khả quy. Giả sử f khả quy trênQ[x], tức là ta có biểu diễn f (x) = f1(x) f2(x) với f1, f2 ∈Q[x], có bậc lớn hơn hoặc bằng 1. Đặt f1(x) = a1 b1 g1(x), f2(x) = a2 b2 g2(x) voi aibi tối giản và g1(x), g2(x) nguyên bản. Khi đó f (x) = f1(x) f2(x) = a1a2 b1b2 g1(x)g2(x) = p q g1(x)g2(x) với (p,q) = 1. Do đó f ∈ Z[x] nên mọi hệ số của khai triển tích g1(x)g2(x) đều là bội số của q. Suy ra đa thức tích g1(x)g2(x) không nguyên bản. Điều này trái với kết quả của Bổ đề Gauss. Vậy f bất khả quy trên Q[x]. 2.1.1 Đa thức với hệ số thực và phức Cho một đa thức với hệ số thực thì chưa chắc đa thức đó có nghiệm trong trường số thực, cụ thể đa thức x2+1 không có nghiệm trong trường số thực. Dưới đây ta sẽ thấy mọi đa thức bậc n với hệ số phức có đúng n nghiệm phức. Để chứng minh ra hãy đưa vào các bổ đề sau đây: Bổ đề 2.1.4. Mọi đa thức với hệ số thực có bậc lẻ có ít nhất một nghiệm thực. 38 Chứng minh. Giả sử f (x) = anxn+an−1xn−1+ ....+a0, an 6= 0 và n lẻ. Qua giáo trình giải tích ta biết rằng với những giá trị dương và âm của x, khá lớn về giá trị tuyệt đối, hàm số f (x) có các dấu trái nhau. Vậy có những giá trị thực của x, a và b chẳng hạn, sao cho. f (a) 0. Mặt khác hàm số f (x) là liên tục, vì vậy có một giá trị c của x, nằm giữa a và b, sao cho f (x) = 0. Bổ đề 2.1.5. Mọi đa thức bậc hai ax2+ bx+ c, với hệ số phức, bao giờ cũng có hai nghiệm phức. Bổ đề 2.1.6. Mọi đa thức bậc lớn hơn 0 với hệ số thực có ít nhất một nghiệm phức. Định lí 2.1.7. Mọi đa thức bậc lớn hơn 0 với hệ số phức có ít nhất một nghiệm phức. Chứng minh. Giả sử f (x) là một đa thức bậc n> 0 f (x) = a0+a1x+ ....+anxn với hệ số phức. Đặt f (x) = a0+a1x+ ....+anxn với các ai là các liên hợp của các ai với i= 0, . . . ,n. Xét đa thức g(x) = f (x) f (x). Ta có g(x) = b0+b1x+ ....+b2nx2n Với bk = ∑ i+ j=k aia j, k = 0,1, ....,2n. 39 Vì bk = ∑ i+ j=k aia j = bk nên các hệ số bk là thực. Theo Bổ đề 2.1.6 g(x) có ít nhất một nghiệm phức z= s+ it, g(z) = f (z) f (z) = 0. Do đó hoặc f (x) = 0 hoặc f (z) = 0. Nếu f (z) = 0, f (z) = a0+a1z+ ....+anzn = 0. thì a0+a1z+ . . .anzn = a0+a1z+ . . .+anzn = 0, tức là f (z) = 0. Như vậy hoặc z hoặc z là nghiệm của f (x). Hệ quả 2.1.8. Các đa thức bất khả quy của vành C[x], C là trường số phức, là các đa thức bậc nhất. Chứng minh. Các đa thức bậc nhất là bất khả quy. Giả sử f (x) là một đa thức của C[x] có bậc lớn hơn 1. Theo Định lí 2.1.7, f (x) có một nghiệm phức c. Vậy f (x) có một ước thực sự x− c, do đó f (x) không bất khả quy. Hệ quả 2.1.9. Mọi đa thức bậc n> 0 với hệ số phức có n nghiệm phức. Hệ quả 2.1.10. Các đa thức bất khả quy của R[x], R là trường số thực, là các đa thức bậc nhất và các đa thức bậc hai ax2+bx+ c với biệt số b2−4ac< 0. Chứng minh. Các đa thức bậc nhất và các đa thức bậc hai với biệt số âm rõ ràng là những đa thức bất khả quy của R[x]. Giả sử p(x) là một đa thức bất khả quy của R[x] với bậc lớn hơn một. Vậy p(x) không có nghiệm thực. Theo Định lí 2.1.7, p(x) có một nghiệm phức z và p(x) chia hết cho đa thức bậc hai với hệ số thực. g(x) = x2− (z+ z)x+ zz. 40 Đa thức g(x) không khả nghịch và là ước của phần tử bất khả quy p(x), vậy g(x) phải là liên kết của p(x), tức là p(x) = ug(x), 0 6= u ∈ R. Ta có điều cần chứng minh. 2.1.2 Đa thức bất khả quy của vành Q[x] Đối với trường số thực R và trường số phức C, vấn đề xét xem một đa thức đã cho của vành R[x] hay C[x] có bất khả quy hay không rất đơn giản, nhưng trong vành Q[x] với Q là trường số hữu tỉ thì vấn đề phức tạp hơn nhiều. Đối với các đa thức bậc hai và ba của Q[x], việc xét xem có bất khả quy hay không được đưa về việc tìm nghiệm hữu tỉ của đa thức đó. Các đa thức bậc hai và bậc ba của Q[x] là bất khả quy khi và chỉ khi chúng không có nghiệm hữu tỉ. Đối với các đa thức bậc lớn hơn ba thì vấn đề phức tạp hơn nhiều. Chẳng hạn đa thức x4+2x2+1= (x2+1)2 rõ ràng không có nghiệm hữu tỉ nào, nhưng nó có một ước thực sự x2+ 1 , vậy không phải là bất khả quy. Ta đã biết, mọi đa thức f (x) với hệ số hữu tỉ đều có thể viết dưới dạng f (x) = b−1g(x) trong đó b là một số nguyên khác 0, g(x) là một đa thức với hệ số nguyên. Trong vành Q[x], f(x) và g(x) là liên kết vậy f (x) là bất khả quy khi và chỉ khi g(x) là bất khả quy. Do đó tiêu chuẩn Eisenstein mà ta đưa ra dưới đây để xét một đa thức của Q[x] có bất khả quy hay không là tiêu chuẩn cho các đa thức với hệ số nguyên. Bổ đề 2.1.11. Nếu f (x) là một đa thức với hệ số nguyên có bậc lớn hơn 0 và f (x) không bất khả quy trong Q[x], thì f (x) phân tích được thành một tích những đa thức bậc khác 0 với hệ số nguyên. Trong nghiên cứu các đa thức bất khả quy, Tiêu chuẩn Eisenstein sau đây là đặc biệt quan trọng. 41 Định lí 2.1.12 (Tiêu chuẩn Eisenstein). Cho f (x) ∈ Z[x], deg f (x) = n, f (x) = a0xn+a1xn−1+ . . .+an. Nếu có số nguyên tố p thỏa mãn ba điều kiện (1) a0 không chia hết cho p; (2) a1,a2, . . . ,an chia hết cho p; (3) an không chia hết cho p2. thì đa thức f (x) bất khả quy trên Q[x]. Chứng minh. Giả sử f (x) có những ước thực sự trongQ[x] theo Bổ đề 2.1.11, f (x) có thể viết f (x) = g(x)h(x), trong đó g(x) = b0+b1x+ . . .+brxr, bi ∈ Z, 0< r < n, h(x) = c0+ c1x+ . . .+s xs, ci ∈ Z, 0< s< n. Ta có a0 = b0c0, a1 = b1c0+b0c1, . . . ak = bkc0+bk−1c1+ . . .+b0ck, an = brcs. Theo giả thiết p chia hết a0 = b0c0; vậy vì p là nguyên tố nên hoặc p chia hết cho b0 hoặc p chia hết cho c0. Giả sử p chia hết b0 thế thì p không chia hết c0, vì nếu thế thì p2 sẽ chia hết a0=b0c0, trái với giả thiết, p không thể chia hết mọi hệ số 42 của g(x), vì nếu thế thì p sẽ chia hết an = brcs, trái với giả thiết. Vậy giả sử bk là hệ số đầu tiên của g(x) không chia hết cho p. Ta hãy xét ak = bkc0+bk−1c1+ . . .+b0ck, trong đó ak,bk−1, . . . ,b0 đều chia hết cho p. Vậy bkc0 phải chia hết cho p. Vì p là nguyên tố, ta suy ra hoặc bk chia hết cho p, hoặc c0 chia hết cho p, mâu thuẫn với giả thiết về bk và c0. 2.2 Một số bài toán điển hình Bài toán 2.2.1 (IMO 1993). Cho n ∈ N và n> 1. Chứng minh rằng đa thức f (x) = xn+5xn−1+3 bất khả quy trên Z[x]. Lời giải. Với n= 2 có f (x) = x2+5x+3 thì bất khả quy trên Z[x]. Xét n≥ 3. Giả sử f (x) = g(x)h(x) với g(x), h(x)∈Z[x] và có bậc lơn hơn hoặc bằng 1. Do degg(x)+degh(x) = n≥ 3 nên suy ra trong hai số degg(x) và degh(x) có một số lơn hơn 1. Vì f (0) = 3 là số nguyên tố nên hoặc |g(0)|= 1 hoặc |h(0)|= 1. Giả sử g(x) = xk+b1xk−1+ . . .+bk với k > 1 và |g(0)|= 1. Gọi a1,a2, . . . ,ak là các nghiệm (nói chung là nghiệm phức) của g(x). Khi đó ta có g(x) = (x−a1)(x−a2) . . .(x−ak). Vì |g(0)|= 1 nên |a1a2 . . .ak|= 1. (2.1) 43 Do g(ai) = 0 nên f (ai) = 0, với i ∈ {1, . . . ,k}. Nhân các đẳng thức đó lại và sử dụng (2.1) ta được |(a1+5)(a2+5) . . .(ak+5)|= 3k (2.2) Mặt khác ta có g(−5) = |(a1+5)(a2+5) . . .(ak+5)| và 3= f (−5) = g(−5)h(−5) nên |(a1+5)(a2+5) . . .(ak+5)| nhận giá trị 1 hoặc 3. Điều này trái với (2.2) vì k > 1. Từ đó ta có ta có điều phải chứng minh. Bài toán 2.2.2 (Dự tuyển IMO). Cho đa thức f (x) =∏ni=1(x−ai)−2 với n≥ 3 và a1,a2, . . .an là các số nguyên đôi một khác nhau. Chứng minh rằng nếu f (x) khả quy trên Z[x] thì n= 3. Lời giải. Giả sử f (x)= g(x)h(x) với g(x), h(x)∈Z[x] và degg= p≥ q= degh≥ 1. Ngoài ra, giả sử các hệ số bậc cao nhất của g và h đều bằng 1. Trong phép đặt cho x = ai ta sẽ nhận được f (ai) = g(ai)h(ai) =−2. Như vậy ta có (g(ai),h(ai)) ∈ {(−2,1),(2,−1),(1,−2),(−1,2)} Vậy với mọi i, ta có g(ai)+ h(ai) = +1 với ít nhất n2 giá trị của i, hoặc g(ai)+ h(ai) =−1 với ít nhất n2 giá trị của i. Giả sử g(ai)+ h(ai) =1 với k giá trị của i ∈ {1, . . . ,n}. Rõ ràng k < n (vì đa thức g(x)+h(x)−1 6= 0 và có bậc nhỏ hơn thực sự n. Giả sử k ≥ n2 ≥ 2, vì nếu k < n2 thì xét đa thức g(x)+h(x)+1. Giả sử g(ai)+h(ai) = 1 với i ∈ {1, . . . ,k}, nếu không như vậy thì ta đặt lại thứ tự của chỉ số. Suy ra g(x)+h(x)−1= (x−a1)(x−a2) . . .(x−ak)w(x) với w(x) ∈ Z[x]. 44 Cho x= ak+1 với chú ý g(ak+1)+h(ak+1) =−1 ta được −2= (ak+1−a1)(ak+1−a2) . . .(ak+1−ak)w(ak+1). Thế nhưng trị tuyệt đối của một tích gồm ít nhất là bốn thừa số nguyên tố khác nhau và khác 0 thì l

Các file đính kèm theo tài liệu này:

  • pdfluan_van_da_thuc_trong_cac_bai_toan_thi_hoc_sinh_gioi.pdf
Tài liệu liên quan